Minimalpolynom

Neue Frage »

SD Karte Auf diesen Beitrag antworten »
Minimalpolynom
Meine Frage:
Sei K ein Körper und sei A Element aus M(nxn)(K) eine invertierbare Matrix. Sei ma das Minimalpolynom von A. Beweisen Sie, dass es ein Polynom f Element aus K[T] mit Grad(f)<Grad(ma) so gibt, dass A^-1=f(a) ist.

Hat jemand einen Ansatz?

Meine Ideen:
Also ich habs schon mit dem Adjunktensatz probiert, aber nichts Gutes rausbekommen.
galoisseinbruder Auf diesen Beitrag antworten »

Ich hätt´ ne Idee, hab aber keine Ahnung ob das die vorgesehene ist.
berechnen (Du darfst noch zeigen warum dass X ist).
Bezout liefert dann eine Darstellung .

Edit1 : merke gerade, dass ich zu kompliziert gedacht hab. Der ggT ist nach einer Zeile schon fertig. Wir können also ohne ggT formulieren:
und dann auf die Darstellung gehen.
Edit2 : Das rote X war mal ein 0, darauf bezieht sich der Kommentar von tmo, dem ich danke, dass er meinen Tippfehler gesehen hat.
tmo Auf diesen Beitrag antworten »

Kleine Anmerkung: Es geht nicht darum zu zeigen, dass der ggT nicht 0 ist, sondern dass der Rest bei der Division nicht 0 ist. Mit anderen Worten ist zu zeigen: .

Dies kann man entweder nur durch die Minimalität des Minimalpolynoms zeigen (ich mag solche Beweise smile ) oder etwas mehr Theorie über Eigenwerte nutzen, denn diese schenkt einem die Behauptung quasi schon.
SD_karte Auf diesen Beitrag antworten »

Geht die Minimalität des Minimalpolynoms nicht aus der Def. hervor? Wie kann man daraus meinen Satz zeigen? Also f(A)=A^-1
SD Karte Auf diesen Beitrag antworten »

Und wo kommt euer X her - was bedeutet das?
galoisseinbruder Auf diesen Beitrag antworten »

Ersetze das X durch T is es dann klar? (ich bevorzuge als Variable X deutlich vor allen anderen)
 
 
Sd Karte Auf diesen Beitrag antworten »

Achso natürlich und wie lautet die Antwort auf meinen 1. Post?
Sd Karte Auf diesen Beitrag antworten »

Also ma(0)!=0 zu zeigen, bekomme ich hin, aber wie ziehe ich dann den Schluss zu meiner Aufgabe?
galoisseinbruder Auf diesen Beitrag antworten »

Was tmo meint ist dieses: Du sollt aus der Minimalität des Minimalpolynoms folgern, dass
Zitat:
.
oder für meine Formulierung

Edit: warst etwas schneller als ich. Zum Finden des f würde ich meinen ersten Post beachten.
Sd Karte Auf diesen Beitrag antworten »

In Ordnung. Das habe ich - da brauche ich keine Hilfe (danke für die Hilfe bis jetzt). Aber wie komme ich von da auf die Behauptung???
galoisseinbruder Auf diesen Beitrag antworten »

Setz mal A ein.
Sd Karte Auf diesen Beitrag antworten »

Na ma(A)=0 und ma(0)=c. Und dann?
galoisseinbruder Auf diesen Beitrag antworten »

Wie wärs mit hier:
Zitat:
.

oder hier:
Zitat:
Sd Karte Auf diesen Beitrag antworten »

Ist dein großes X wirklich das Polynom des Grades 1 mit dem Koeffizienten 1? Und wenn ich einsetze in 1bekomme ich 1=fA oder?
galoisseinbruder Auf diesen Beitrag antworten »

Ja X ist das X aus ;
Wenn Du einsetzt, dann bitte vollständig, also auch in f.
Was wird aus der 1 wenn wir von Polynomen in X zu Summen/Potenzen von matrizen übergehen?
Sd Karte Auf diesen Beitrag antworten »

f(a)=1 oder? Hilf mir mal bitte auf die Sprünge ich muss in 5min los.
galoisseinbruder Auf diesen Beitrag antworten »

Nein, das ist falsch.
Es ist wahrscheinlich sinnvoller Du denkst in Ruhe drüber nach und wir sehens uns dann nochmal an.
Sd Karte Auf diesen Beitrag antworten »

Seh ich auch so. Ich komme noch mal um halb acht oder so. Könnten Sie mir noch erklären, ob ich A als X nehmen soll, oder wie sie "einsetzen" meinen. Oder besser noch wo ich "einsetzen" soll. Danke für die Hilfe.
Sd Karte Auf diesen Beitrag antworten »

Seh ich auch so. Ich komme noch mal um halb acht oder so. Könnten Sie mir noch erklären, ob ich A als X nehmen soll, oder wie sie "einsetzen" meinen. Oder besser noch wo ich "einsetzen" soll. Danke für die Hilfe. (Ist eig. 1=f*ma+g*T => 1=g(a)*(A) => gesuchte f ist g???)
galoisseinbruder Auf diesen Beitrag antworten »

In dem Sinne das die 1 im Matrizenring E ist, ist Deine letzte Anmerkung genau die Aussage auf die ich raus wollte. (in meiner Gleichung sind nicht umsonst die Bezeichnungen f und g vertauscht.)
Leopold Auf diesen Beitrag antworten »

Warum wird das alles so kompliziert formuliert?
Ich würde einfach die Gleichung mit durchmultiplizieren und nach auflösen. Wegen ist das möglich. (Und daß ist, behauptet Sd Karte, sei ihm klar).
Sd Karte Auf diesen Beitrag antworten »

Danke an alle. Beide Wege habe ich verstanden. Dass a0!=0 folgt wiegesagt daraus, dass das Minimalpolynom minimal ist. (Kein Problem, das zu beweisen) Ich denke, wir können den Thread schließen.
Neue Frage »
Antworten »



Verwandte Themen

Die Beliebtesten »
Die Größten »
Die Neuesten »